Last visit was: 26 Apr 2024, 05:52 It is currently 26 Apr 2024, 05:52

Close
GMAT Club Daily Prep
Thank you for using the timer - this advanced tool can estimate your performance and suggest more practice questions. We have subscribed you to Daily Prep Questions via email.

Customized
for You

we will pick new questions that match your level based on your Timer History

Track
Your Progress

every week, we’ll send you an estimated GMAT score based on your performance

Practice
Pays

we will pick new questions that match your level based on your Timer History
Not interested in getting valuable practice questions and articles delivered to your email? No problem, unsubscribe here.
Close
Request Expert Reply
Confirm Cancel
SORT BY:
Kudos
Founder
Founder
Joined: 04 Dec 2002
Posts: 37311
Own Kudos [?]: 72891 [3]
Given Kudos: 18869
Location: United States (WA)
GMAT 1: 750 Q49 V42
GPA: 3
Send PM
Current Student
Joined: 20 Oct 2018
Posts: 184
Own Kudos [?]: 127 [1]
Given Kudos: 57
Location: India
GMAT 1: 690 Q49 V34
GMAT 2: 740 Q50 V40
GPA: 4
Send PM
Founder
Founder
Joined: 04 Dec 2002
Posts: 37311
Own Kudos [?]: 72891 [0]
Given Kudos: 18869
Location: United States (WA)
GMAT 1: 750 Q49 V42
GPA: 3
Send PM
Intern
Intern
Joined: 12 Jan 2018
Posts: 40
Own Kudos [?]: 48 [0]
Given Kudos: 36
Location: India
Concentration: General Management, Marketing
GMAT 1: 680 Q49 V32
WE:Engineering (Energy and Utilities)
Send PM
Re: V10-32 [#permalink]
Why can't the answer be E?
Intern
Intern
Joined: 20 Nov 2017
Status:MBA Student
Posts: 22
Own Kudos [?]: 126 [0]
Given Kudos: 107
Location: India
Concentration: Strategy, Marketing
GPA: 3.9
WE:Consulting (Consumer Electronics)
Send PM
Re: V10-32 [#permalink]
ManasviHP wrote:
Why can't the answer be E?


Because 'E' says that the second is a questionable contention that the resident supports with additional evidence. The second statement is not an argument that is given along with the evidence. Since the second statement is the evidence given by the residents, option 'D' is the correct answer.
IIM School Moderator
Joined: 05 Jan 2015
Status:So far only Dreams i have!!
Posts: 386
Own Kudos [?]: 352 [0]
Given Kudos: 214
WE:Consulting (Computer Software)
Send PM
Re: V10-32 [#permalink]
Please somebody explain option D & E. Not able to understand the expected answer for this question. It would be very helpful if somebody can throw some light on it. Thanks..
Intern
Intern
Joined: 02 May 2020
Posts: 9
Own Kudos [?]: 0 [0]
Given Kudos: 10
Send PM
Re: V10-32 [#permalink]
I selected A. I need help understanding how the City Offical's bolded statement is a conclusion rather than evidence. Is it because that sentence begins with 'clearly'?
GMAT Club Bot
Re: V10-32 [#permalink]
Moderator:
Math Expert
92932 posts

Powered by phpBB © phpBB Group | Emoji artwork provided by EmojiOne